Mecânica dos fluidos/Exercícios resolvidos/D2: diferenças entre revisões

[edição não verificada][edição não verificada]
Conteúdo apagado Conteúdo adicionado
MaskedAce (discussão | contribs)
Sem resumo de edição
MaskedAce (discussão | contribs)
Sem resumo de edição
Linha 104:
 
 
<center><math>\frac{1}{3} \; \left. ln \left( \frac{2 p_{\infty}}{\rho} \;+\; 3u \right) \right|_0^u \;=\; -\; \left. \frac{}{} ln(R) \right|_a^R</math></center>
 
 
<center><math>\frac{1}{3} \; ln \left( \frac{\frac{2 p_{\infty}}{\rho} \;+\; 3u}{\frac{2 p_{\infty}}{\rho}} \right) \;=\; ln \left( \frac{a}{R} \right)</math></center>
 
 
<center><math>\left( 1 \;+\; \frac{3 \rho u}{2 p_{\infty}} \right) ^ {\frac{1}{3}} \;=\; \frac{a}{R}</math></center>
 
 
<center><math>u \;=\; \frac{2 p_{\infty}}{3 \rho} \; \left( \left( \frac{a}{R} \right) ^ 3 \;-\; 1 \right)</math></center>
 
 
Pode-se, então, encontrar o tempo decorrido através de uma integração
 
 
<center><math>\Delta t \;=\; \int dt \;=\; \int_a^0 \frac{dR}{v} \;=\; \int_a^0 \frac{dR}{\sqrt{u}}</math></center>
 
 
<center><math>\Delta t \;=\; \int_a^0 \frac{dR}{\sqrt{\frac{2 p_{\infty}}{3 \rho} \; \left( \left( \frac{a}{R} \right) ^ 3 \;-\; 1 \right)}} \;=\; \left( \frac{2 p_{\infty}}{3 \rho} \right) ^{- \; \frac{1}{2}} \; \int_a^0 \left( \left( \frac{a}{R} \right) ^ 3 \;-\; 1 \right) ^{- \; \frac{1}{2}} \; dR</math></center>